Clinical Chem/Urinanalysis Midterm

Pataasin ang iyong marka sa homework at exams ngayon gamit ang Quizwiz!

What enzymes catalyzes the transfer of an amino group to alpha-ketoglutarate with the formation of oxaloacetate and glutamate?

Aspartate aminotransferase (AST)

Which one of the following enzymes catalyzes the transfer of an amino group to alpha ketoglutarate with the formation of oxaloacetate and glutamate?

Aspartate aminotransferase (AST)

Which one of the following enzymes catalyzes the transfer of an amino group to alpha- ketoglutarate with the formation of oxaloacetate and glutamate

Aspartate aminotransferase (AST)

Which one of the following enzymes catalyzes the transfer of an amino group to aplha-ketoglutarate with the formation of oxaloacetate and glutamate?

Aspartate aminotransferase (AST)

When pyuria is detected in a sediment, the slide should be carefully checked for the presence of

Bacteria

The renal artery branches off into the afferent arteriole, which:

Becomes the capillary tuft within the glomerulus

An autosomal recessive genetic disease that leads to copper deposition in the body tissues. The accumulation of copper damages the liver, brain and eyes

Wilson's Disease

Which substance found in urine is considered pathologic?

Yeast

Which of the following pairings of stool appearance and cause does not match

Yellow-green: barium sulfate

Because your immunoassay analyzer is down for maintenance, you have performed an immunoinhibition assay to determine the relative presence of CK isoenzymes in an individual who has presented with acute upper body pain. The assay reveals a strong activity after inhibition of CK-M subunits using a monoclonal antibody. This result indicates

a nonspecific result due to possible presence of CK-B subunits.

In regard to the expression of enzyme activity, a katal is:

a unit that describes the amount of enzyme that will catalyze a mole of substrate in 1 second

Reagent strip tests for ketones measure primarily

acetoacetic acid

A reaction rate assay measures enzyme _____________; an endpoint assay measures enzyme ___________.

acivity, activity

The major non-glucose-reducing substance present in serum is:

acorbic acid

A specimen with a negative nitrate reaction and a positive leukocytes reaction that has WBCs, WBC casts, and no bacteria in the sediment will be seen in cases of

acute interstitial nephritis

Elevated levels of amylase and lipase are an indication of

acute pancreatitis

Secretion of renin and aldosterone is induced by low blood pressure and volume. Renin is synthesized in the _____ and aldosterone is made in the _____

adrenal gland; brain

C-reactive protein (CRP) is considered an acute phase protein (APP) because its concentration in blood is increased quickly during inflammation. An example of a negative APP would be

albumin

The plasma protein that serves to transport a large number of compounds including bilirubin, calcium, drugs

albumin

The protein section of the urine reagent strip is most sensitive to

albumin

Ghost red blood cells are seen in urine that is

alkaline and dilute

The electrophoretic pattern shown in a densitometric tracing showing marked alpha-1 globulin decrease, slightly decreased albumin, and slight increase in alpha-2 most likely indicates:

alpha-1 antitrypsin

The serum protein that is analyzed to determine the presence of emphysema is

alpha1-antitrypsin

An example of a posttranslational modification of an enzyme that produces an enzyme isoform would be

alteration of carbohydrate side chains

A white precipitate in a urine specimen with a pH of 7.5 would most probably be caused by

amorphous phosphates

Which of the following enzymes requires chloride (Cl) as an activator?

amylase

The proteins that are deposited in tissues as aggregates and that form b-sheet structures with eventual damage to the tissues are referred to as:

amyloid

The build up of fluid in a body cavity is called

an effusion

The presence of C-reactive protein in the blood is an indication of:

an inflammatory process

At present, the major diagnostic usefulness of tartrate-resistant acid phosphatase type 5b is in its use as a marker of conditions with

an osteolytic component.

Cessation of urine flow is defined as

anuria

Analyzing protein in cerebrospinal fluid is done to:

assess the permeability of the blood -brain barrier.

The quaternary structure of a protein is the

association of other peptide chains with the main protein

A urine specimen comes to the laboratory 7 hours after is obtained. It is acceptable for culture only if the specimen has been stored

at 4-7 degrees Celsius

The acid-base properties of amino acids are important in their function. Amino acids and proteins are both referred to as ampholytes. This means

at neutral pH, they have balanced positive and negative charges.

Urine samples should be examined within 1 hour of voiding because

bacterial contamination will cause alkalinization of the urine

Increased CSF lactate is found in

bacterial meningitis

A clean-catch urine is submitted to the laboratory for routine urinalysis and culture. The routine urinalysis is done first, and 3 hours later, the specimen is sent to the microbiology department for culture. The specimen should

be rejected due to the time delay

A reagent strip area impregnated with stabilized, diazotized 2,4-dichloroaniline will yield a positive reaction with

bilirubin

Which of the following urine results is most apt to be changed by prolonged exposure to light

bilirubin

Amniotic fluid is evaluated using a Liley graph and change in absorbance at 450 nm. What is being evaluated and why

bilirubin, which increases in HDN

Activators increase the rates of enzyme-catalyzed reactions. In some cases, these activators interact with the nonenzymatic component of the reaction such as the substrate. However, in most cases the activator

binds to the enzyme similar to the enzyme/substrate combination.

Activators increase the rates of enzyme-catalyzed reactions. In some cases, these activators interact with the nonenzymatic component of the reaction such as the substrate. However, in most cases the activator:

binds to the enzyme similar to the enzyme/substrate combination.

The component of an electrophoresis system that carries the applied current and establishes the pH at which electrophoresis is performed is the

buffer

Most proteins are synthesized :

by the hepatic parenchymal cells.

Tiny, colorless, dumbbell-shaped crystals were found in an alkaline urine sediment. They most likely are

calcium carbonate

Which of the following crystals may be found in acidic urine

calcium oxalate

Plasma urea concentration

can increase because of decreased renal perfusion

Plasma urea concentration:

can increase because of decreased renal perfusion

At a pH of 4.0, the

carboxyl and amino groups of an amino acid become protonated, causing the amino acid to have a net positive charge.

Urea is produced from:

catabolism of proteins and amino acids

Creatininase reactions for creatinine assessment

catalyze the conversion of creatinine to creatine

All of the statements below about clinically significant phosphatases are true except they:

catalyze the transfer of amine groups to ketoacids

All of the statements below about the clinically significant phosphatases are true except they:

catalyze the transfer of amine groups to ketoacids.

In an electrophoresis system, the _____ is the negative electrode

cathode

Failure to observe RBC casts in a urine specimen can be caused by

centrifuging an unmixed specimen

Which serum protein should be measured in a patient suspected of having Wilson's disease?

ceruloplasmin = alpha-2 globulin; decreased in Wilson's dz

A 46-year-old patient visits her physician with a complaint of chest pain, blood in her urine, and oliguria. She states that these symptoms have gotten worse over the past 2 to 3 months. Urine and blood samples are collected. Urine GFR is calculated tobe 40 mL/min/1.73 m2 and hemoglobin is 8 g/dL. Urine protein was elevated, with the presence of red blood cell casts. Upon review of her health history, it was noted that she was a cigarette smoker with hypertension. The most likely diagnosis in this case would be:

chronic kidney disease (CKD)

The volume of plasma from which a substance is completely removed by the kidneys per unit of time is the definition of:

clearance

The appearance of normal CSF is

colorless and clear

Urine specific gravity is an index of the ability of the kidney to

concentrate the urine

The amount of creatinine produced each day is fairly constant and is related to the muscle mass. In health, the concentration of creatinine in the bloodstream is also fairly constant.

concentration in plasma is increased by a high-protein diet

When performing a routine urinalysis, the technologist notes a 2+ protein results. He should

confirm with the acid precipitation test

The use of several enzymatic reactions linked together to provide a means of measuring the activity of the first enzyme or the concentration of the initial substrate is referred to as a(n):

consecutive enzymatic reaction.

Urine reagent strips should be stored in a(n)

cool dry area

In the biuret reaction or protein quantification:

copper ions complex with peptide bonds in proteins.

Serum proteins or protein complexes that precipitate at temperatures lower than normal core body temperature and that, if precipitated in tissues, result in vasculitis and ischemic injury to peripheral tissues are referred to as:

cryoglobulins

A sweat chloride >60 mEq/L (60 mmol/L) is indicative of

cystic fibrosis

Which of the following is an abnormal crystal described as a hexagonal plate

cystine

Which pair does not match with respect to amniotic fluid

dark green - hemolytic disease of the newborn

If the viscosity of the support medium used in a protein electrophoresis system is too high (thick), the migration rate will

decrease

Hyperproteinemia is caused by

dehydration

The most common genetic defect associated with cystic fibrosis is called

delta-F508

Any condition, such as extreme temperature or extremes of pH, which changes the shape of the enzyme protein structure generally causes loss of enzymatic activity. This is referred to as

denaturation

Peritoneal lavage is used to

detect intra-abdominal bleeding in blunt injury

The following glucose tolerance test results are indicative of what state? Glucose values as assayed by glucose oxidase method are fasting= 130 mg/dL; 60 minutes= 225 mg/dL; and 120 minutes= 205 mg/dL.

diabetes

A 45-year-old man visits his physician with complaints of insatiable thirst, sudden onset of fatigue, polydipsia, and polyuria. Laboratory results indicate a normal fasting blood sugar. Serum sodium was slightly elevated. Urine was clear and had low specific gravity (hypotonic). The most likely cause of these symptoms and laboratory results would be:

diabetes insipidus

A patient has glucosuria, hyperglycemia and polyuria. These findings are most consistent with

diabetes mellitus

Glycation of glomerular basement membranes and capillaries is the main nephropathic effect from:

diabetes mellitus

Polyuria is usually correlated with

diabetes mellitus

Mr. Smith Glucose 682 mg/dL, serum ketones 4+

diabetic ketoacidosis

During embryonic and fetal development, changes in isoenzyme distribution patterns are common. These changes are thought to result from

differential expression and changes in the relative activities of gene loci within developing cells

The possibility of detecting glitter cells is associated with urine that is

dilute

In regard to factors that govern the rate of an enzymatic reaction, first-order reaction kinetics occur at that part of the reaction during which the rate of the reaction is

directly proportional to the substrate concentration

In regard to factors that govern the rate of an enzymatic reaction, first-order reaction kinetics occur at that part of the reaction during which the rate of the reaction is:

directly proportional to the substrate concentration

Analyses of free light chains, particularly immunoassays that measure the ratio of free k- to free l-light chains, are sensitive indicators of:

disorders of immunoglobulin synthesis such as multiple myeloma.

To prepare the reagent used in confirmatory protein testing, a technician would

dissolve 3 g sulfosalicylic acid in 100 mL of water

Osmolality is a measure of

dissolved particles, including ions

A urine's specific gravity is directly proportional to its

dissolved solids

Which of the following aids in differentiating a spherical transitional cell from a round renal tubular cell

eccentrically-placed nucleus in the renal tubular cell

Known functions of proteins normally present in plasma include all of the following except:

emulsification of fat.

The advantage to using phase microscopy in urinalysis is to

enhance constituents with a low refractive index

The ____ form of glucose if responsible for its reducing properties

enol

Which stain is used to measure sperm viability

eosin nigrosin

Recognized abnormalities of fatty acid catabolism associated with uncontrolled diabetes mellitus include the development of ketones. This ketosis is caused by

excess acetyl-CoA being diverted to form ketone bodies

False results in urobilinogen testing may occur if the urine specimen is

exposed to light

Amniotic fluid is tested for the concentration of lamellar bodies. This test determines

fetal lung maturity (FLM)

Polarized light can often be used to differentiate between

fibers and mucus clumps

The clarity of a urine sample should be determined

following thorough mixing of the specimen

Prior to reporting a red blood cell cast, it is important to observe

free-floating RBCs

A urinalysis performed on a 2-week-old infant with diarrhea shows a negative reaction with the glucose oxidase reagent strip. A copper reduction tablet test should be performed to check the urine sample for the presence of

galactose

The reason for performing a Clinitest on a newborn's urine is to check for

galactose

The most sensitive enzyme indicator for liver damage from ethanol intake is

gamma-glutamyl trnasferase (GGT)

To prepare the reagent used for mucin clot determination of synovial fluid, water is mixed with

glacial acetic acid

Creatinine clearance is used to assess the _____ of a patient.

glomerular filtration rate

An example of a monosaccharide would be:

glucose

Which of the following positive chemical reactions is most closely associated with the presence of yeast in the urine sediment

glucose

A urine specimen is analyzed for glucose by a glucose oxidase reagent strip and a copper reduction test. If both results are positive, which of the following interpretations is correct

glucose is present

beta-glucose + O2 -----> D-glucose-d-lactone + H2O2

glucose oxidase

Which of the following is the best indicator of Reye syndrome for CSF (hepatic encephalopathy)

glutamine

The major storage carbohydrate in animals is _____, which is located most abundantly in _____

glycogen; the liver

The breakdown of glycogen stores in the liver to increase serum glucose is referred to as:

glycogenolysis

Conversion of glucose into lactate or pyruvate is referred to as:

glycolysis

Which of the following inhibits glycolysis and glucose uptake by muscle cells and causes a rine in blood glucose concentrations?

growth hormne

The chromogen for the fecal occult blood test is

gum guaiac

The plasma protein that demonstrates a decreased serum concentration during an intravascular hemolytic disorder and is used to identify this occurrence is

haptoglobin

The isoelectric point (pI) of a protein is the pH at which the protein

has no net charge

At a physiological pH of 7.4, most amino acids

have no net charge.

A reagent strip test for blood has been reported positive. Microscopic examination fails to yield red blood cells. This patient's condition can be called

hemoglobinuria

The reference method for glucose determination is:

hexokinase

This patient's LDL cholesterol is:

high

When employing the urine reagent strip method, a false-positive protein result may occur in the presence of

highly alkaline urine

When a coenzyme binds to an enzyme, the enzyme portion of the resulting molecule is referred to as the

holoenzyme

False-positive results can occur for fecal occult blood due to the ingestion of

horseradish

Which of the following casts is most likely to be found in healthy people

hyaline

In the colorimetric determination of cholesterol, using the enzyme cholesterol oxidase, ____________ is the agent that oxidizes the colorless organic compound, 4-aminoantipyrine, to a pink complex.

hydrogen peroxide

The largest source of error in creatinine clearance tests is

improperly timed urine specimens

Ketones in urine are due to

incomplete fat metabolism

Upon standing at room temperature, a urine pH typically

increases

Zero-order kinetics occurs during the beginning of an enzyme-catalyzed reaction when a substrate concentration is high and the rate of the reaction is _____ on the _____ concentration

independent; substrate

Pilocarpine iontophoresis refers to the specific process of

inducing sweat

Immunofixation electrophoresis

involves exposure of electrophoresed proteins to antisera against different immunoglobulin heavy and light chains with resultant precipitation

In capillary electrophoresis, hydrodynamic sample injection

involves sample injection into the capillary by applying a positive pressure to the sample inlet

The theory of electrophoresis states that:

ionized chemical species having an electrical charge move toward either the cathode (negative electrode) or the anode (positive electrode).

Endosmosis

is the movement of solvent along with highly hydrated ions to produce the driving force in capillary electrophoresis

A type of electrophoresis that provides increased resolution in protein separation by allowing the protein to migrate to a point where the protein's charge becomes zero and its migration stops is _____ electrophoresis

isoelectric focusing

Primary gout is distinguished from secondary gout by

its polygenic basis

A 17-year-old girl decided to go on a starvation diet. After 1 week of starving herself, what substance would most likely be found in her urine

ketones

Which of the following enzyme tests is most affected by "hemolysis"

lactate dehydrogenase

Which of the following components are present in serum but not present in the glomerular filtrate

large molecular weight proteins

Bacteria are considered significant in the urine sediment when the

leukocytes is positive

The enzyme that in inhibited by hemoglobin is

lipase

The principal source of serum cholinesterase (CHE) in blood is

liver.

The best way to lower the light intensity of the microscope is to

lower the rheostat

A technologist is having trouble differentiating between red blood cells, oil droplets and yeast cells on a urine microscopy. Acetic acid should be added to the sediment to

lyse the red blood cells

The creatinine clearance is reported in

mL/min

Which of the following is NOT considered a coenzyme?

magnesium

Activators that are required for normal CK activity include:

magnesium only

Activators that are required for normal CK activity include:

magnesium only.

Milky urine from a 24-year-old woman would most likely contain

many white blood cells

After receiving a 24-hour urine for quantitative total protein analysis, the technician must first

measure the total volume

When determining the activity of an enzyme in serum as in a bisubstrate reaction, measurement of two different substances can be made. One measurement determines the decrease in substrate concentration acted upon by the enzyme and the other

measures the increase in the concentration of the second product formed.

A brown-black urine would most likely contain

melanin

One international unit is defines as one ______ of substrate converted to product per ______

micromole, minute

The primary clinical usefulness of assessing alpha-fetoprotein (AFP) as a tumor marker is for:

monitoring therapy in patients with liver cancer

The recommended assay, because of its specificity and sensitivity for the measurement of the activity of the CK-MB isoenzyme, is the

monoclonal sandwich immunoassay.

The main advantage of capillary electrophoresis over standard gel electrophoresis is

more efficient heat dissipation for better separation.

Which semen result is abnormal

motility of 1.0

"A malignant neoplasm of clone of plasma cells that diffuses throughout the bone marrow and that is diagnosed in part by identification of paraprotein in blood or urine is referred to as:

multiple myeloma.

A malignant neoplasm that is composed of plasma cells that exhibit monoclonality and are diffusely present in the bone marrow is referred to as:

multiple myeloma.

Ammonium sulfate was added to red urine. The urine had a positive reaction for blood, but no RBCs were seen on microscopic examination. After centrifugation the supernatant fluid is red. The abnormal color is caused by

myoglobin

Red urine may be due to

myoglobin

Regarding enzyme kinetics, the Michaelis-Menten plot of the relationship between reaction velocity and substrate concentration is correctly expressed as which one of the following formulae?

n = Vmax[S]/Km + [S]

The functional unit of the kidney is the

nephron

Microalbumin can be measured by a random urine collection. An increased microalbumin is predictive of

nephropathy

A 62-year-old patient with hyperlipoproteinemia has a large amount of protein in his urine. Microscopic analysis yields moderate to many fatty, waxy, granular and cellular casts. Many oval fat bodies are also noted. This is most consistent with

nephrotic syndrome

A 10-year-old girl visits her pediatrician with the complaint of bone pain. Following enzyme studies, these results were obtained: ALP 3.5 higher than upper reference limit; NTP normal. These results are consistent with a diagnosis of

normal bone growth

A 10-year-old girl visits her pediatrician with the complaint of bone pain. Following enzyme studies, these results were obtained: ALP 3.5 higher than upper reference limit; NTP normal. These results are consistent with a diagnosis of:

normal bone growth

The volume of urine excreted in a 24-hour period by an adult patient was 300 mL. This condition would be termed

oliguria

Varying different factors and studying their effects on an enzymatic reaction rate in the assessment of most favorable reaction conditions for an enzyme assay is referred to as

optimization

2. An ampholyte is a(n):

organic compound that contains both amino and carboxyl functional groups.

The most accurate test to determine renal concentration is

osmolarity

Tartrate-resistant acid phosphatase (TR-ACP) that is present in unhemolyzed serum is synthesized mainly in

osteoclasts

Following ingestion of ethylene glycol (antifreeze), numerous crystals are found in the urine. The shape of these crystals is

oval/dumbbell

Match the enzyme with its correct enzyme classification. Creatine kinase Lactate dehydrogenase Alkaline phosphatase Pyridoxal phosphate Magnesium Calcium

oxi-lac/ lacta-creat/ hydro-alk phosph/ pyrido-asp/ mag-creat/calc-amylase

A reaction in which NAD+ or NADP+ is formed from reagent NADH or NADPH is _______ and may be measure by a _______ in absorbance at 340nm.

oxidase, decrease

Uricase is used in the assay of uric acid to

oxidize uric acid to form allantoin

The most valuable initial aid for the identification of crystals in urine specimen is

pH

When identifying urinary crystals, which reagent strip result is most important

pH

Which of the following would be affected by allowing a urine specimen to remain at room temperature for 3 hours before analysis

pH

The isoelectric point of an amino acid or protein is defined as the:

pH at which the amino acid or protein has no net charge

Factors governing the rate of enzyme reactions are:

pHtemperaturesubstrate concentrationcofactorsALL CORRECT

A patient with uncontrolled diabetes mellitus will most likely have

pale urine with a high specific gravity

Ascites is collected by

paracentesis

A positive result for bilirubin on a reagent strip should be followed up by

performing an Ictotest

While performing an analysis of a baby's urine, the technologist notices the specimen to have a "mousy" odor. Of the following substances that may be excreted in urine, the one that most characteristically produces this odor is

phenylpyruvic acid

In electrophoresis, the best support medium to use because it is thermostable, strong, transparent, and eliminates endosmosis due to the lack of a charge is

polyacrylamide

Which of the following methods for lipoprotein electrophoresis depends on charge and molecular size?

polyacrylamide gel

Urine from a 50-year-old man was noted to turn dark red on standing. This change is caused by

porphyrins

Telephoning a positive Clinitest result on a new born

postanalytical factor

Immobilized enzymes are used analytically in various electrochemical techniques. The use of an ion-selective electrode that is coated with an enzyme that produces ions when placed in a substrate solution is a type of:

potentiometric measurement.

Collecting a timed urine specimen

preanalytical

Preventive maintenance schedules on the centrifuge used to spin down urine samples is a _________ component of quality assurance.

preanalytical

Rejection of a contaminated specimen

preanalytical factor

The addition of sulfosalicylic acid for a qualitative turbidimetric protein determination of a urine specimen is done to:

precipitate a protein in the specimen that might be missed by albumin urine dipstick methods

The finding of hemosiderin laden macrophage in a CSF sample indicates

previous hemorrhage

The major cellular origin of acid phosphatase activity in serum is:

prostate

The portion of a nephron considered the most metabolically active and that is involved in the reabsorption of 60% to 80% of the glomerular filtrate and that secretes 90% of hydrogen ion excreted by the kidney is the:

proximal tubule

Upon microscopic examination, an individual's urine shows many bacteria, white blood cells, and cellular casts composed of polymorphonuclear leukocytes. It is likely that this individual has

pyelonephritis

The presence of leukocytes in urine is known as

pyuria

The method of choice for performing a specific gravity measurement of urine following administration of x-ray contrast dyes is

reagent strip

Damage to the glomerulus would be suspected when the urine sediment contains

red blood cell casts

IgG

reflects past infection

The major artery that expands into the capillary bed that forms the glomerulus is the:

renal artery

A 21-year-old woman had glucose in her urine with a normal blood sugar. These findings are most consistent with

renal glycosuria

Glycosuria may be due to

renal tubular dysfunction

In a specimen with a large amount of bilirubin, which of the following sediment constituents would be most noticeably bile-stained

renal tubular epithelial cell casts

Spherical urothelial cells may be confused with

renal tubular epithelial cells

Oval fat bodies are

renal tubular epithelial cells that contain lipids

On a serum protein electrophoresis, if the albumin band appears distorted and large, the likely cause is

sample overload.

To avoid falsely elevated spinal fluid cell counts

select an aliquot from the last tube collected

Albumin concentration in blood can be analyzed in an empty blood collection tube containing no anticoagulant. This means that you are analyzing

serum that has been obtained when no anticoagulants are mixed with whole blood and then centrifuged.

All of the following conditions are associated with hyperuricemia except

severe hepatocellular disease

The greatest source of CK is:

skeletal muscle

Use of a refractometer over a urinometer is preferred due to the fact that the refractometer uses

small volume of urine and compensates for temperature

To prepare a solution appropriate for quality control of the refractometer, a technician should use

sodium chloride with a specific gravity of 1.022

Which of the following reagents is used to react with ketones in the urine

sodium nitroprusside

Urine osmolality is related to

specific gravity

A microscopic examination of urine sediment reveals ghost cells. These red blood cells are seen in urine with a

specific gravity of <1.007

What cell is most commonly associated with vaginal contamination

squamous

Clue cells are a form of

squamous epithelial cell

Which of the following contaminants has a dimpled center and will polarize

starch

Which of the following statements may be associated with the activity of insulin?

stimulates glycogenisi in the liver

Granular casts found in the urine of a football player admitted to the hospital with a broken leg occurring during the game can be the result of

strenuous exercise

The confirmatory test for a positive protein result by the reagent strip method uses

sulfosalicylic acid

T/F: Methods used as screening tests for cystic fibrosis include

sweat conductivity

The element of protein structure that involves the folding of the polypeptide in three dimensions and the formation of disulfide bonds between cysteines is the _____ structure

tertiary

The interaction of the amino acid side chains with the arrangement of the a-helices and b-sheets to form a three-dimensional protein structure is called the _____ structure of the protein

tertiary

An abdominal fluid is submitted from surgery. The physician wants to determine if this fluid could be urine. The technologist should

test for urea and creatinine

True isoenzymes, which are multiple forms of an enzyme that possesses the ability to catalyze an enzyme's characteristic reaction, are formed by:

the existence of more than one gene locus coding for the structure of the enzyme protein

To distinguish between a clump of WBCs and a WBC cast, it is important to observe

the presence of a cast matrix

"As a separation technique, serum protein electrophoresis depends on:

the ratio of the protein's charge to hydrodynamic size.

All of the statements below about the clinically significant phosphatases are true except

they catalyze the transfer of amine groups to ketoacids

Amniocentesis should be performed to

to confirm a high maternal serum alpha-fetoprotein (MSAFP)

Statin drugs lower cholesterol by competitive inhibition of the cholesterol-synthesizing enzyme HMG-CoA reductase. A competitive inhibitor binds

to the active center of the enzyme, thereby causing the Km to increase.

A CSF was collected from a 5-year-old with a fever, and 3 tubes were transported to the lab. Tube 1 had 50000 RBC/mL and 48 WBC/mL. Tube 3 had 10 RBC/mL and 0 WBC/mL. What is the most likely explanation for the discrepancy

traumatic tap

Regarding measurement of a property related to substrate concentration such as fluorescence production, the assay that, although more technically demanding, theoretically provides the most accurate measurement of enzyme activity is the

two-point kinetic method

Which of the following crystals appear as fine, silky needles

tyrosine

Which of the following crystals is seen in an amber urine with a positive bilirubin

tyrosine

Hyaline casts are usually found

under subdued light

The usefulness of analyzing lipoprotein-associated phospholipase A2 for the identification of individuals at risk for future cardiovascular events is because of its:

upregulation in atherosclerotic lesions, particularly complex plaque

After warming, a cloudy urine clears. This is due to the presence of

urates

The major nonprotein nitrogen degradation product of endogenous purines is:

uric acid

The yellow color of urine is primarily due to

urochrome pigment

Which of the following cells is most likely to be seen in the urine sediment following a catheterization procedure

urothelial cell

Regarding enzyme kinetics, the Michaelis-Menten plot of the relationship between reaction velocity and substrate concentration is correctly expressed as what formula?

v= Vmax[S]/Km+ [S]

Bilirubinuria may be associated with

viral hepatitis

Microscopic analysis of a urine specimen yields a moderate amount of red blood cells in spite of a negative result for occult blood using a reagent strip. The technologist should determine if this patient has taken

vitamin C

A 2-year-old child had a positive urine ketone. This would most likely be caused by

vomiting

Normal urine primarily consists of

water, urea and sodium chloride

Which of the following cast most frequently appears to have brittle consistency

waxy

Which of the following casts is most indicative of end-stage renal disease

waxy

Glitter cells are a microscopic finding of

white blood cells in hypotonic urine

The causative agent of the clinical diagnosis in question 7 is:

β-hemolytic streptococci

Select the most sensitive marker for alcoholic liver disease

γ-Glutamyltransferase (GGT)

How does pH alter an enzymatic reaction rate?

By affecting key amino acids in the enzyme protein at the active center and other sites

Men have, on average, higher HDL cholesterol levels but lower total cholesterol & triglyceride levels than women.

FALSE

T/F: In an enzyme immunoassay such as ELISA, the specificity of the labeled enzyme is the most important aspect of the measurement

FALSE

The ideal specimen for triglyceride analysis is serum.

FALSE

True or False? Pre-beta (VLDL) lipoproteins migrate further toward the anode on polyacrylamide gel than they do on cellulose acetate or agarose.

FALSE

True or false? In a "kinetic" or "rate" reaction, one spectrophotometric measurement is taken at the end of an incubation period.

FALSE

True or false? In proximal renal tubular acidosis, tubular cells cannot maintain a pH gradient and urine remains above pH 6.5

FALSE

True or false? Increased levels of apolipoprotein A-I are associated with increased risk of coronary artery disease.

FALSE

A negative urine pregnancy test performed on a random specimen may need to be repeated using a

First morning specimen

A urine specific gravity is ordered for a patient, and it is performed on the first morning specimen and the last evening specimen. The specific gravity for both urine samples is 1.010; this indicates:

Fixation of the urine solute composition as a result of renal failure

Polyuria and nocturia are common features of chronic renal failure as a result of:

Fixation of the urine solute composition in the filtrate

Increased enzyme activity is typically observed in serum or plasma

Following injury or damage to tissue

Which of the following methods of measuring enzyme activity utilizes an increase in absorbance of 340 nm as an indicator reaction?

Forward LD reaction

Mr. Smith, a regular client at the laboratory, comes in for a UA. The technician comments that she hasn't seen Mr. Smith in quite awhile. He tells her that he has been at the University Hospital, undergoing chemotherapy for cancer. He says that he has had a rough time and has lost a lot of weight, but he feels much better thanks to a new medication, MESNA. His appetite is back, and he has been eating regularly. When she tests Mr. Smith's urine, the technician notes that the ketone pad immediately turns purple. Results from the Clinitek show 4+ ketones. She suspects a false-positive result caused by:

Free sulfhydryl groups

Which one of the following is the correct formula for determining glomerular filtration rate?

GFR = ([urine concentration of the substance] × volume)/[plasma concentration of the substance]

Which is the most sensitive enzyme for monitoring chronic alcoholic liver disease

GGT

Which of the following enzymes detects very early changes in liver cell damage?

GGT

Which of the following are the most common causes of acute pancreatitis?

Gallstones or obstructing tumorAcute alcohol ingestion or drug use

In a hypotonic urine, RBCs have what type of appearance?

Ghost cell

A genetic liver disease caused by reduced activity of the enzyme glucoronyl transferase, which conjugates bilirubin to a water-soluble form. The most common cause of hereditary hyperbilirubinemia

Gilbert Syndrome

The patient brings in a 24-hour urine collection at 8 AM and states that upon collection of the first morning specimen today, he wasn't quite awake, forgot, and voided a small portion into the toilet but quickly stopped and then properly collected the remainder into the container. You should:

Give the patient another container and instruct him to recollect a new 24-hour specimen

If a physician requests a creatinine clearance on an individual, what is the physician attempting to determine?

Glomerular filtration rate (GFR)

A 50-year-old female is admitted to the hospital. She states that she has not been feeling well for the past week and that she started running a fever several days ago. Examination and testing reveal bacterial endocarditis and renal insufficiency. The UA results are:

Glomerulonephritis

A type 1 diabetic who is in the early stages of the disease is taken to the emergency room with symptoms of hypoglycemia.LACK of what endogenous hormones might be the cause of this

Glucagon

By what mechanism does glucagon promote hyperglycemia?

Glucagon stimulates glucose synthesis by gluconeogenesis

The formation of glucose from compounds like fatty acids, glycerol, and amino acids is called:

Gluconeogenesis

The technician performs a UA and a copper reduction test on a 19-month- old child. The reagent strip shows a positive result for glucose; the copper reduction test also produces a positive result. Which of the following is most likely present in the urine?

Glucose

The technician reads over the urinalysis test results that just printed off the Clinitek and notes that the specific gravity was 1.039. Which urinalysis test results should the technician check for correlation?

Glucose

Which enzyme is measured in whole blood?

Glucose-6-phosphate dehydrogenase

A diabetic patient is prescribed a daily regimen of insulin. Which of the following laboratory procedures would be of the most value in determining the degree of glucose control over a 2-month period?

Glycated hemoglobin

In the Szasz method for gamma glutamyltransferase (GGT), the typical receptor molecule is:

Glycyl glycine

Which substrate concentration is needed to achieve zero-order conditions?

Greater than 99 × Km

Oxidation of bilirubin to biliverdin in urine results in which of the following urine colors?

Green

The lipoprotein that contains the greatest amount of protein is called:

HDL

A 75 year old man was brought by his daughter to the emergency department. He was in a confused state, trembling, sweating, with a rapid pulse, pigheadedness, and complaining of hunger and epi-gastric discomfort. His daughter claims that he does not drink alcohol and that he takes oral hypoglycemic drugs to control his diabetes. What is the likely cause of his symptoms?

He has possibly taken too much of his medication and is suffering from hypoglycemia

Aspartate aminotransferase is elevated in diseases involving the:

Heart muscle

Urinalysis on a patient with severe back and abdominal pain is frequently performed to check for

Hematuria

Which of the following laboratory results would point to a diagnosis of acute nephritic syndrome in an individual who exhibits hypertension and edema?

Hematuria, sodium retention, decreased GFR, proteinuria

Concerning specimen requirements for total serum calcium determination, which of the following anticoagulants is acceptable?

Heparin

An autosomal recessive genetic disease that causes an abnormal accumulation of iron in organs, leading to organ injury and toxicity. (Iron overload)

Hereditary Hemochromatosis

A 35-year-old man presents to his physician with complaints of pain in his spine and large joints that has been progressively worsening over the past 6 months. He also points out that his ears are getting darker in color. He sheepishly reveals that, to conserve water, he does not always flush the toilet after urination, and he has noticed that when he voids, the urine is yellow, but when he comes back later, it has changed to a much darker color. The physician orders a gas chromatography-mass spectrometry analysis on the patient's urine. The results are positive for:

Homogentisic acid

Healthy individuals may have which of the following in their urine, as a normal finding, after strenuous exercise?

Hyaline and finely granular casts

What type of morphologic change results in the accumulation of a homogeneous eosinophilic extracellular material in the glomeruli?

Hyalinization of glomeruli

In homeostatic regulation of plasma acid-base concentrations, sodium is both actively and passively exchanged in the tubules for which one of the following ions?

Hydrogen

Serum lipase catalyzes:

Hydrolysis of triglyceride to fatty acids and glycerol

As blood flows through the glomerulus, the plasma ultrafiltrate collects in Bowman's space as a result of:

Hydrostatic pressure

An 80-year-old female with diabetes presents to the ED with heart failure. You might expect to see which of the following conditions confirmed by blood glucose and UA testing?

Hyperglycemia and no glucosuria

The most common glomerular disease worldwide is

IgA nephropathy

A tumor of plasmacytoid cells that is diagnosed as Waldenström macroglobulinemia produces a paraprotein that is an

IgM

HBsAg NegativeAnti-HBc PositiveAnti-HBs Positive

Immune due to natural infection

HBsAg NegativeAnti-HBc NegativeAnti-HBs Positive

Immune to hepatitis vaccine

An increase in Bence-Jones protein in urine typically indicates the presence of paraproteins. What is Bence-Jones protein?

Immunoglobulin light chain fragments

The technician reports the following results on a routine UA for an 89- year-old female being given her annual physical examination: The patient collected a CCMS specimen on site at 2 PM, and the urine was tested at 3 PM. The results indicate which of the following?

Improper collection of the midstream clean catch specimen

Protein error of indicators in the protein reagent test pad may result in false positive readings:

In highly buffered or alkaline urine (≥9.0)

The four types of renal tubular acidosis involve all of the following pathogeneses except:

Increased secretion of calcium carbonate, resulting in hypocalciuria

Renin is released in response to all of the following except:

Increased sodium

All of the following chemical changes may occur in unpreserved urine specimens except:

Increased urobilinogen

What effect does glucagon have on blood glucose value?

Increases

All of the following are principal roles of the secretory process except:

Maintenance of fluid volume

The technician is logging urine color and clarity for four specimens he had running on the Clinitek. He notes an abnormal orange color for specimen #3. All test pads for specimen #3 are positive. Upon microscopic examination, the technician reports a negative result for all cellular and crystal elements. Specimens #1 and #4 test negative for all constituents, and no microscopic examinations are performed. Specimen #2 tests positive for nitrites and leukocytes only; that microscopic examination reveals WBCs and bacteria. The next course of action the technician should take is:

Manually retest specimen #3

Which of the following inherited diseases gives the urine a characteristic mousy or musty odor?

Maple syrup urine disease

A patient showing signs of polydipsia, polyuria, polyphagia, and glyco- suria most likely is not producing enough:

Insulin

Prior to identifying an RBC casts, all of the following should be observed except

Intact RBCs in the cast

Even distribution of blood in all tubes

Intracranial hemorrhage

Xanthrochromic supernatant

Intracranial hemorrhage

Osmolality better reflects the kidneys' ability to concentrate urine because specific gravity:

Is influenced by the presence of glucose and protein

Which of the following plays an important role in the formation of renal calculi?

Isohydruria

Which statement is not true about cellular morphology in urine specimens?

It degenerates as pH decreases.

All of the following are true about the refractive index as a measure of specific gravity except:

It is a measurement of the mass of solutes per volume of solution.

A healthy 27-year-old pregnant woman in the second trimester sees her obstetrician for a routine visit. Blood and urine samples are obtained. Total serum protein and protein electrophoresis are performed on the blood sample and total urine protein is also analyzed. The results of the serum protein electrophoresis indicate a substantial increase in the -globulin region of the electrophoretogram. Which of the -globulins might this be and why

It is likely alpha1-fetoprotein due to fetal development

A healthy 27-year-old pregnant woman in the second trimester sees her obstetrician for a routine visit. Blood and urine samples are obtained. Total serum protein and protein electrophoresis are performed on the blood sample and total urine protein is also analyzed. The results of the serum protein electrophoresis indicate a substantial increase in the a-globulin region of the electrophoretogram. Which of the a-globulins might this be and why?

It is likely alpha1-fetoprotein due to fetal development

The primary producer of renin is a small endocrine organ located in the:

Juxtaglomerular apparatus

Which of the following ultrafiltrate substances is passively reabsorbed into the peritubular capillary blood?

K+

The term critical value refers to a(n):

Life-threatening test result

"The refractive index of a solution is affected by all of the following except:"

Light scattering by particles in the solution

Serum bilirubin is unstable because of

Light sensitivity

The function of the basement membrane in the glomerulus is to:

Limit passage of filtrate substances by size

Which one of the following serum enzymes produced by the exocrine pancreas is the most specific in a differential diagnosis for detection of acute pancreatitis?

Lipase

What statements concerning lipase is incorrect?

Lipase is a small enzyme that can be detected in urine

Which one of the following statements concerning lipase is incorrect

Lipase is a small enzyme that can be detected in urine

Liquefaction of a semen specimen should take place within

Liquefaction of a semen specimen should take place within

The proximal and distal convoluted tubules are connected by the:

Loop of Henle

The progressive loss of the GFR in chronic renal failure clinically presents when the patient shows which of the following?

Loss of 80% to 85% of normal renal function

A 45-year-old patient who received a kidney transplant 2 months ago presents to the ED with a temperature of 100° F, chills, aches, headache, dizziness, oliguria, and a weight gain of 6 lb since yesterday. The technician notes that the predomi- nant WBC in the microscopic examination is:

Lymphocytes

Heart tissue contains what CK isoenzymes?

MM and MB

In addition to a Chemical Hygiene Plan, the laboratory must provide employees access to _____ under the employee "right to know."

MSDS

A positive glucose test result on a urinalysis indicates:

Maximal tubular reabsorptive capacity exceeded

Reference Range for ALT

May-35

In a continuous-monitoring assessment of an enzyme reaction rate, which one of the following is the preferable measurement

Measurement of the increasing concentration of product

A 50-year-old male presents to his physician, complaining of having had edema, general malaise, fatigue, and anorexia for the past several weeks. The physical examination reveals no other abnormal presentations, except a reported weight gain of 20 pounds in the past 3 days. The clinical history reports no previously existing pathologic conditions. The blood pressure, pulse, and temperature are normal. Blood work and a UA are ordered. The findings include an elevated blood urea nitrogen to creatinine ratio, a decreased creatinine clearance, and a decreased serum protein.

Membranous glomerulonephritis

The laboratory receives a random urine specimen obtained from a 17-year- old female teenager, who needs the UA as part of her sports physical examination. The test results are:

Menstrual contamination

Diabetic patients must routinely have their urine checked to monitor their renal status. For this purpose, the technician performs which test?

Microalbumin reagent strip test

Clarity of urine provides a rapid quality check for which of the following?

Microscopic examination

Which of the following provide(s) the proximal tubular cells with maximal surface area for reabsorption activities?

Microvilli

The pathogenesis of all of the following forms of primary glomeru- lonephritis is antibody mediated except:

Minimal change disease

A 45-year-old woman is seen in the clinic for her annual physical exami- nation. Her general health is good, she is taking no medications, and she notes no physical complaints other than that her weight is creeping up and she has little time for exercise. A new technician, fresh out of college, performs the UA. She then asks another technician to double-check her results, because the chemical analysis does not correlate with the microscopic examination. A repeat UA confirms the chemical analysis results. A repeat microscopic examination is performed as well:

Misidentified mucous threads

What statements regarding creatinine is correct?

Most intra-individual variation in serum creatinine is due to muscle mass

Mrs. Smith has the following chemistry results.CK: moderately increasedLD: moderately increasedLD1 increasedThe probable diagnosis is:

Myocardial infarction

A 16-year-old boy is brought to the ED unconscious and unresponsive. His blood alcohol level is three times the normal limit. His friends report that the last time they saw him conscious was 8 hours ago, around 5 AM. Later that morning, they saw him on the floor in the family room, but they thought he was just sleeping and left him alone. In the afternoon, unable to rouse him, they called 911. The patient is catheterized, and the urine color is brown. The physician is concerned that the urine contains:

Myoglobin

A fresh brown urine may indicate the presence of:

Myoglobin

Several enzymatic triglycerides methods measure the production or consumption of ______________.

NADH

A 37-year-old woman who thinks she might be pregnant visits her physician with a complaint of upper right quadrant pain. Blood samples are obtained. Increased enzyme activity was seen for ALP, AST, lactate dehydrogenase (LD), and GGT. The ALP was 10 times higher than the upper reference limit. A hepatitis panel and a pregnancy test had negative results. Gel electrophoresis was done to separate ALP isoenzymes, with a dense liver band and what appeared to be a diffuse placental band. If this woman's physician has ruled out pregnancy, activity of what other serum enzyme could be measured to differentiate between hepatobiliary disease and liver disease?

NTP

If you were asked by your laboratory director to set up an enzyme profile (group of tests) specifically to assess hepatobiliary (biliary obstruction) disease, which one of the following sets of enzymes would be most appropriate

NTP, ALP, and g-glutamyltransferase (GGT)

If you were asked by your laboratory director to set up an enzyme profile (group of tests) specifically to assess hepatobiliary (biliary obstruction) disease, which one of the following sets of enzymes would be most appropriate?

NTP, ALP, and γ-glutamyltransferase (GGT)

If used correctly, personal protective equipment (PPE) protects the health care worker from bloodborne pathogens from all of the following types of exposures except:

Needle sticks

Urine formation occurs as the plasma ultrafiltrate passes through the renal structures in the following order:

Nephron, calyces, renal pelvis, ureters, bladder

Based on your answer to question 10, the patient is most likely suffering from idiopathic onset of:

Nephrotic syndrome

Glomerular disease associated with lipiduria is known as:

Nephrotic syndrome

Hematuria, proteinuria (>3.5 g/dL), RBC casts, and oliguria are the principal features of:

Nephrotic syndrome

Laboratory tests are performed on a post-menopausal, 57-year-old woman as part of an annual physical examination. The patient's 2-hour post-prandial glucose is 220 mg/dL and serum insulin is 15 mU/ml (normal = 6-26). Based on this information, the patient would be classified as:

Non-insulin-dependent diabetes mellitus (NIDDM)

In the reciprocal plot pictured below, the solid line indicates a normal enzyme reaction with no inhibition, and the dotted line indicates a decrease in Vmax and no change in Km. This plot is an example of which type of inhibition

Noncompetitive

In the reciprocal plot pictured below, the solid line indicates a normal enzyme reaction with no inhibition, and the dotted line indicates a decrease in Vmax and no change in Km. This plot is an example of which type of inhibition?

Noncompetitive

Why do values obtained with the Jaffe reactions for creatinine measurement overestimate creatinine concentration by ~20%?

Noncreatinine chromogens in the reaction that react with the picrate in the end-point Jaffe methods overestimate true plasma creatinine concentration

Which factor will not affect urinalysis results?

None of the above

Which personnel are exempt from adherence to Standard Precautions?

None of the above

In light of the answers to questions 12 and 13, which of the following is the most likely cause of this patient's condition?

Nontraumatic rhabdomyolysis

Which one of the following statements regarding creatinine is correct?

Normal plasma creatinine does not always indicate normal kidney function

The osmolality of an 8 AM urine specimen in a fluid deprivation test was greater than 800 mOsm/kg, which indicates:

Normal renal function

A separation technique that can detect proteins in a complex mixture with the use of antibodies directed against a protein of interest is

Northern blotting

The most abundant complement protein component that is common to all three pathways of complement is

C3.

A phosphotransferase enzyme that has increased activity when injury, inflammation, or necrosis of skeletal muscle occurs is

CK

When myocardial infarction occurs, the first enzyme to reach peak elevation is:

CK

Which of the following enzymes is a transferase

CK

What statements is incorrect concerning creatine kinase (CK)?

CK catalyzes the conversion of creatine and ADP to creatine phosphate and ATP

Which of the following statements is incorrect concerning creatine kinase (CK

CK catalyzes the conversion of creatine and ADP to creatine phosphate and ATP.

Guidelines for developing written safety policies and procedures required by OSHA are available from:

CLSI

The tau isoform of transferrin is a carbohydrate-deficient protein found only in

CSF

Decreased CSF protein can be found in

CSF leakage

What calculation is used to determine if there is a breach in the blood-brain barrier

CSF/serum albumin index

Which type of crystal is found in an acetic urine?

Calcium oxalate

The sediment of a urine specimen with a reagent strip glucose of 250 mg/dL (13.8 mmol/L) and a pH of 5.5 is ideal for the presence of

Candida albicans

During 3 glucose tests an hour apart on a patient, capillary blood is substituted for venous blood for one of the timed samples. What results would you expect for the sample, as compared to results expected for venous blood drawn at the same time?

Capillary blood will yield higher results than venous blood

When using the glass slide and coverslip method, which of the following might be missed if thecoverslip is overflowed

Casts

Which of the following are reported as number per LPF

Casts

The patient is a 3-month-old infant with a possible bladder infection. The best method of urine collection for routine urinalysis testing and culture is:

Catheterization, urethral

Which urine collection method can be used to determine whether kidney disease affects one or both kidneys?

Catheterized ureters

Leaving a reagent strip in the specimen for too long will

Cause reagents to leach from the pads

Acidic urine specimens provide a stable environment for which of the following urine constituents?

Cellular elements

Prior to performing a Gram stain in CSF, the specimen must be

Centrifuged

Following a negative postvasectomy wet preparation, the specimen should be

Centrifuged and reexamined

Aerosol risks in the laboratory setting include all of the following except:

Centrifuging capped tubes

Reagent strip SG indirectly estimates urine density on the basis of:

Charged solutes

The primary reason for performing a Clinitest is to

Check for newborn galactosuria

After an extremely busy night collecting and performing STAT tests, you find a routine urine specimen in the drop basket. You check the collection time and note that the specimen is over 2 hours old. You call the floor and request a new specimen because:

Chemical constituents may be altered, producing invalid results.

A urine crystal that is not considered clinically significant is:

Cholesterol

Anemia, hyperphosphatemia, hypocalcemia, and ongoing azotemia are most indicative of

Chronic renal failure

HBsAg PositiveAnti-HBc PositiveAnti-HBc (IgM) NegativeAnti-HBs Negative

Chronically Infected

The most likely cause for serum/plasma to appear "milky" is the presence of:

Chylomicrons

_____________ are the largest, least dense lipoproteins, produced by the intestine and responsible for a plasma sample's cloudiness after a meal, where they can float to the top of the sample.

Chylomicrons

In stage liver disease accompanied by fibrosis, scarring, and loss of organ function:

Cirrhosis

Proper collection of a midstream clean catch urine specimen requires the patient to:

Cleanse the area, void into the toilet, stop, collect the middle portion into a container, and void the remainder into toilet

The technician receives the following results on a urine specimen from a 1-month-old patient. What additional testing, if any, is required?

Clinitest

While examining a microscopic sediment specimen, the technician sees 1 to 3 squamous epithelial cells that appear soft and finely granular with shaggy edges. He reports:

Clue cells

A specimen with a strong ammonia odor and a heavy white precipitate when it arrives in the laboratory may require

Collection of a fresh specimen

Crystals associated with severe liver disease include all of the following except

Cystine

One of the most common causes of Fanconi's syndrome is:

Cystinosis

An active, vibrant, 85-year-old female is brought to the clinic by her daughter, who states that her mother has been experiencing mental confusion and distress. No other signs or symptoms are reported. The physical examination notes no abnormalities; temperature, pulse, and blood pressure all are normal. A urinalysis is ordered, which produces the following results:

Cystitis

Intralaboratory assessment is useful in monitoring analytical components by:

Determining the accuracy and precision of technicians' performance

A 35-year-old male patient has complained of polyuria and polydipsia. A 24-hour urine collection yields a volume of 5200 mL. Urinalysis reveals negative test results across the board. The physician suspects that the patient may be suffering from:

Diabetes insipidus

Which condition produces a pale urine with a high specific gravity?

Diabetes mellitus

Why might a creatinine level be requested on an amniotic fluid

Differentiate amniotic fluid from maternal urine

Standard Precautions are infection control practices designed to prevent all of the following except:

Disease transmission only for patients put in contact precautions

Which one of the following components of the renal system is most important for regulation of plasma electrolytes and acid-base balance?

Distal convoluted tubule

A decrease in the quality of laboratory testing can be pinpointed by:

Documentation

The principle of the reagent strip test for pH is the

Double indicator reaction

Upon review of the urinalysis printout, the technician notes a urine pH of 9.0 on one specimen. The other test results were negative, except for nitrites, which tested positive. The next step is:

Double-check the time of collection, transport, and storage

An elderly trauma patient presents to the emergency department with sudden anuria. The most likely finding is:

Dramatically decreased renal perfusion

A random clean catch urine specimen for cytology studies requires:

Drinking 24 to 32 ounces of water each hour 2 hours before urine collection

A 21-year-old man with nausea, vomiting, and jaundice has the following laboratory findings

Early hepatitis

A 75-year-old man has excruciating flank pain on the left side. The doctor suspects a kidney stone. UA findings of blood and calcium oxalate crystals, along with a CT scan, confirm the diagnosis. The physician treats the patient and advises him to:

Eat a vegetarian diet

You operate a laboratory that receives many patient samples from the renal clinic next door. These patients have serious kidney problems, with causes that range from diabetes and IgA nephropathy to Fanconi syndrome. Would you expect to see elevated or decreased total urine protein from the patients that go to this clinic

Elevated

A practicing physician at a university student healthcare facility often recommends to student patients that they take large doses of vitamin C (ascorbic acid) to fend off colds during the semester. As director of the chemistry laboratory that serves this facility, you notice a trend in that many of this physician's students' serum creatinine levels are unusually high. You are currently using a type of analysis based on the Jaffe reaction. What might be the cause of the elevated values?

Elevated ascorbic acid in serum produces a Jaffe-like chromogen in this analysis that can be interpreted as increased serum creatinine

Which one of the following is a correct statement describing a property of an enzyme?

Enzymes are protein catalysts that decrease the activation energy of a chemical reaction

What is a correct statement describing a property of an enzyme?

Enzymes are protein catalysts that decrease the activation energy of a chemical reaction.

A 32-year-old woman presents to the ED with fever, oliguria, hematuria, and a rash. She reports that she has a sinus infection and is currently taking penicillin. The physician suspects that the patient has acute interstitial nephritis (AIN); she therefore orders which tests to be performed on the patient's urine specimen?

Eosinophil count using Hansel stain

The most common pathogen in UTIs is:

Escherichia coli

The technician is busy trying to cover several departments, because she is the lone technician on the third shift. She receives several routine urinalysis specimens, STAT blood work for chemistry and hematology, and a STAT cross-match; in addition, the emergency department is hopping. How will the technician prioritize her workload?

Evaluate the color and clarity of the specimens and then refrigerate

The reagent pad for blood detection is impregnated with:

Tetramethylbenzidine and peroxide

The colligative properties of a sample depend on the:

Number of solute particles

The Occupational Health and Safety Act of 1970 established formal regulation of safety and health for all employees, regardless of employer; these regula- tions are administered by:

OSHA

The highest levels of alkaline phosphatase are seen in:

Obstructive liver disease

The net filtration pressure of 10 mm Hg is the result of which three pressures working in concert for the formation of a plasma ultrafiltrate in Bowman's space?

Oncotic, hydrostatic, afferent arteriole

Several urine specimens come into the laboratory around lunchtime. The technician checks the times on the specimens and notes that she can go to lunch and get back in time to perform the UA without having to refrigerate the specimens. Upon her return, she finds that one of the specimens is now black. The pH test result is 8.0, so the technician:

Orders a test for homogentisic acid

Measures of freezing point depression can be used to calculate all of the following except:

Osmolality

Uncontrolled diabetes mellitus will result in:

Osmotic diuresis

Which of the following would not show an increase in alkaline phosphatase?

Osteoporosis

Why is heparin the only acceptable anticoagulant to use when obtaining blood samples for ALP analysis

Other anticoagulants contain inhibitors of magnesium, which is a required cofactor for ALP activation

Cells found in urine sediment that contain highly refractile droplets are:

Oval fat bodies

In the spectrophotometric determination of AST, malic dehydrogenase catalyzes the conversion of:

Oxalacetate to malate

The technician receives a urine specimen from the OB floor and notes an orange-brown color. She suspects:

Oxidation of urobilinogen

Prostatic acid phosphatase measurement has been replaced by:

PSA

Which marker can be used to identify a body fluid as semen

PSA

The daily total urine volume reflects all of the following except:

Patient's surface area

Transmission-Based Precautions apply to:

Patients colonized with infectious agents

Based on the answer to question 11, the technician will:

Perform confirmatory testing

External quality assurance (QA) is a means to measure the:

Performance relative to other laboratories

The microscopy technique that provides more detailed visualization of translucent or low-refractile components and living cells is:

Phase-contrast microscopy

Upon dipping the chemical reagent strip into orange urine, the technician notes that the entire reagent strip is orange. He suspects the presence of:

Phenazopyridine

The three secretory mechanisms that maintain blood pH and involve the secretion of H+ include all of the following except:

Phosphate reabsorption

What is the distinguishing clinical feature of the type of porphyria that results in the accumulation of porphyrins?

Photosensitivity

Oxidation of porphobilinogen to porphobilin causes the urine to take on a:

Pink color

Which of the following are metabolically active cells with phagocytic abilities that prevent occlusion of the filtration barrier by macromolecules?

Podocytes and mesangium cells

A patient with diabetes mellitus may excrete an elevated number of osmoles of solute per day, resulting in:

Polydipsia

Conditions of water diuresis or solute diuresis result in:

Polyuria

Upon opening a urine specimen container, you are almost knocked over by the smell of ammonia. You check the time of collection and note that it is not within the 2-hour time frame. You expect which of the following results:

Positive nitrites

While performing QC checks on an open reagent bottle, the technician notes that no open date/time is written on the bottle. The technician should address this as a(n):

Preanalytical error; the technician should discard the bottle, obtain a new one, and start over

When elevated amounts of bilirubin are presented to the liver with elevated serum unconjugated bilirubin, what type of jaundice occurs?

Prehepatic

While processing a urine specimen, the technician notes yellow foam that does not dissipate upon standing. He suspects the presence of:

Protein and bilirubin

All of the following will cause false-positive protein reagent strip values except

Proteins other than albumin

A finding of increased numbers of casts and/or the presence of abnormal casts must be accompanied by a finding of:

Proteinuria

What is the first feature of nephrotic syndrome that results in manifestation of the other four classic signs

Proteinuria

A 65-year-old male is admitted to the hospital suffering from azotemia, hyperkalemia, metabolic acidosis, and oliguria. The patient is currently being treated with an aminoglycoside for a urinary tract infection. The physician suspects toxic tubular necrosis and expects to see which of the following listed on the urinalysis report?

Proximal renal tubular cell

Glucose is actively transported back into the peritubular capillary blood in the:

Proximal tubule

A 60-year-old man diagnosed with paroxysmal nocturnal hemoglobinuria can tell from the color of his urine when he needs a transfusion. When the urine gets really dark and stays dark, he knows it is time for a transfusion. He drops off a random urine sample at the laboratory at 0730, indicating that he had collected it 30 minutes earlier.

Prussian blue stain

As the only technician on the third shift, you receive a routine urine specimen that was collected at 0300 hours. It is now 0400 hours. You have been called to the emergency department for STAT blood draws on two trauma patients. You decide to:

Put the specimen in the refrigerator for later testing

What products are formed from the forward reaction catalyzed by ALT?

Pyruvate and glutamate

Which of the following products are formed from the forward reaction catalyzed by ALT?

Pyruvate and glutamate

Interlaboratory testing provides additional quality control checks on all of the following except:

QC material

Timed urine specimens are used for:

Quantitative chemical analysis

As the technician is scanning for casts, she notes several hyaline-looking casts with a few yellowish cells that appear to be embedded in the matrix. She also notes several muddy brown casts. After closer inspection, the technician reports out which of the following?

RBC cast

Production of bilirubin occurs in the __________, and conjugation of bilirubin to glucuronide occurs in the _____________.

RES; liver

The presence of which substance is considered iatrogenic?

Radiographic dye

While performing UAs, the technician ran controls and began testing the specimens. He ran out of reagent strips and needed one more to complete the run. He opened a new bottle and:

Ran controls on the new bottle and then continued the run

Which type of glomerulonephritis may develop as a result of a systemic disease, such as systemic lupus erythematosus, or after an infection?

Rapidly progressive glomerulonephritis

An individual presents to his physician with generalized weakness and fatigue. Blood is collected and an elevated WBC count with lymphocytosis is noted. Serum protein is moderately decreased, but the urine reagent dipstick does not indicate proteinuria. Upon confirmatory testing with a precipitation test, the urine protein is 4+. Based on other symptoms, the physician suspects multiple myeloma. What might be the cause of the discrepancy in urine protein values?

Reagent dipsticks respond mostly to urine albumin and not to other proteins

Which urine color is not the result of drug metabolites?

Red-purple

First morning specimens provide the ideal specimen for all of the follow- ing reasons except:

Reduced number of epithelial cells

HBsAg: Hepatitis B surface antigen

Reflects current infection; a person with a positive HBsAg result should be considered infectious

Anti-HBs IgM: Hepatitis B surface antibody

Reflects recent infection (within last 6 months)

IgM

Reflects recent or current infection (within last 6 months)

A 37-year-old woman who thinks she might be pregnant visits her physician with a complaint of upper right quadrant pain. Blood samples are obtained. Increased enzyme activity was seen for ALP, AST, LD, and GGT. The ALP was 10 times higher than the upper reference limit. A hepatitis panel and a pregnancy test had negative results. Gel electrophoresis was done to separate ALP isoenzymes, with a dense liver band and what appeared to be a diffuse placental band. If there is not a pregnancy, what is the name given to the observed isoenzyme band that migrates in the same region as the placental isoenzyme

Regan isoenzyme

A urine specimen is received in the laboratory with the patient's name, the date, and the time of collection on the lid of the container. You should:

Reject the specimen as unlabeled

The reagent strip test pad for specific gravity produces a color change by measuring the:

Release of protons by ionic solutes, resulting in a decreased pH

Timed urine collections are useful in measuring the excretion of urine substances affected by diurnal variation, which include all of the following except:

Renal cells

When a solute is filtered but not secreted or reabsorbed by the nephrons, its concentration in the urine is referred to as:

Renal clearance

Ischemic acute tubular necrosis (ATN) may result from all of the follow- ing conditions except:

Renal glycosuria

Vitamin D-resistant rickets is due to the inherited dominant sex-linked disorder:

Renal phosphaturia

Detection of proteinuria in individuals with diabetes is important as a marker for all of the following except:

Renal plasma flow

Peritubular capillaries function in the process of reabsorption and secretion of substances in conjunction with the:

Renal tubules

The technician dips a reagent strip into a urine specimen and then reads it manually, following the manufacturer's specifications. After performing the microscopic examination, he notes that the dipstick color on the edge of the reaction pad for ketones is darker than when he had first read it. The technician:

Reports the original result and discards the strip and other disposable items

The technician receives an unlabeled urine specimen in the laboratory; the technician should promptly:

Request that the specimen be recollected

All of the following are true about waxy casts except they

Require staining to be visualized

Which of the following specimen types is not suitable for bacterial and fungal cultures?

Routine void

The integrity of reagent strips is best assessed by:

Running control materials

A hydrolase enzyme important during digestion that catalyzes the hydrolysis of starches while food is still in the mouth is:

S-type amylase

A hydrolase enzyme important during digestion that catalyzes the hydrolysis of starches while food is still in the mouth is

S-type amylase.

Urine creatinine clearance levels can be underestimated because of which of the following?

Salicylate

All of the following are noted as the filtrate moves through the proximal tubule except:

Secretion of urea

The kidneys play a major role in acid-base balance by:

Selectively excreting acid or alkali

A physician calls the laboratory and asks what lab tests should be requested to assess the acid-base balance function of a patient's renal system. You reply:

Serum bicarbonate and blood pH

Refractometry is a rapid, semiquantitative method of total protein determination. Which of the following will lead to a faulty interpretation of protein values obtained by refractometry?

Serum protein less that 3.5 g/dL

You have been asked what laboratory tests should be requested to assess the electrolyte balance regulatory function of an individual's kidneys. Which of the following is your reply?

Serum sodium and potassium, and arterial blood pH

Plasma proteins (e.g., albumin) are not normally found in the urine filtrate because of which of the following?

Shield of negativity

Would a control sample that has accidentally become diluted produce a trend or a shift in the Levy-Jennings plot

Shift

All of the statements about specific gravity are true except:

Specific gravity relates the volume of urine to the density of an equal volume of pure water.

The first morning UA results on a 60-year-old male suspected of having a bladder infection are: Which of the following is likely to be questionable?

Specific gravity, nitrite, leukocyte esterase, and blood

In a random urine sample from a 50-year-old woman, the technician notes large, free-floating, homogenous fat globules with no intracellular presence. The globules stain orange with Sudan III. The urinalysis results are unremarkable thus far:

Specimen contaminated

A positive nitrite test and a negative leukocyte esterase test is an indication of a

Specimen older than 2 hours

The presence of which type of cell indicates vaginal contamination in a random urine specimen or an improperly collected midstream clean catch urine specimen?

Squamous cell

The stain most commonly used to enhance visualization of urine sediment is:

Sternheimer-Malbin

T/F: Glucose values above 1000 mg/dL, normal or elevated plasma sodium and potassium, slightly decreased bicarbonate, elevated BUN and creatinine, and elevated osmolality are all laboratory findings of a patient with type 2 diabetes in nonketotic hyperosmolar state.

TRUE

True or false? In renal tubular failure, serum levels of B2 microglobulin are decreased and urine levels are increased.

TRUE

The technician records these results on a UA and Clinitest: glucose 4+ on reagent strip and negative for Clinitest. The most likely explanation is:

The Clinitest result indicates that the pass-through effect has occurred.

A hemolyzed serum sample from an individual is assayed for creatinine and a result is obtained that is decreased compared to the normal reference interval. If this individual has a normal glomerular filtration rate, which one of the following statements is true?

The assay must be subject to interference from bilirubin

The technician performs an admission UA on a urine specimen that is beer brown in color and that was obtained from a homeless 25-year-old male hospitalized with hepatitis. The results are:

The bilirubin result is a false negative caused by the presence of nitrite.

A technician performing a microscopic examination on a urine sediment is trying to determine whether he is looking at a large white cell or a renal tubular cell. He reports 2 to 4 renal tubular cells based on which criterion?

The cell is polygonal with one flat edge.

You have just set up the morning protein electrophoresis. When you return 45 minutes later, you stain the gel and notice that the bands did not migrate very far from the point of application. What is the likely explanation?

The current or voltage on the power supply was set too low.

"Which of the following statements regarding enzymatic reactions is true"

The enzyme increases the rate of the reaction

The CSF tube labeled 3 is sent to

The hematology department

In competitive inhibition of an enzyme reaction

The inhibitor binds to the enzyme at the same site as does the substrate

The kidneys play a role in all of the following processes except:

The kidneys play a role in all these processes.

Which of the following statements concerning chylomicrons is false?

The major lipid transported by this lipoprotein is cholesterol

If an individual has a normal GFR and a hyperchloremic normal anion gap metabolic acidosis with a freshly voided early morning urine specimen which has a pH of 6.5, what is the likely diagnosis?

There is likely the onset of distal renal tubular acidosis (RTA)

A specimen is received in a green top tube (heparin) with a request for alkaline phosphatase analysis. The plasma is separated and the test is performed. How will this specimen type affect the ALP results?

There will be no effect, because this is an acceptable specimen for this test

A specimen is received in a green top tube (heparin) with a request for alkaline phosphatase analysis. The plasma is separated and the test is performed. How will this specimen type affect the ALP results

There will be no effect, because this is an acceptable specimen for this test.

Upon receiving a serum sample for protein electrophoresis, which you perform using agarose gel, you note that the specimen is hemolyzed. How would hemolysis affect the banding pattern of this type of electrophoresis?

There would be an increased b-globulin band.

All enzymes are classified to one of six classes based on the reaction they catalyze. Based on this classification, creatine kinase is a member of what enzyme class?

Transferase

All enzymes are classified to one of six classes based on the reaction they catalyze. Based on this classification, creatine kinase is a member of which one of the following enzyme classes?

Transferase

What proteins are important in the regulation of circulating iron

Transferrin

Which of the following proteins are important in the regulation of circulating iron?

Transferrin

Which condition produces a negative urine bilirubin result but also an increased urobilinogen result?

Transfusion reaction

Concentration of blood in tube 1 is greater than in tube 3

Traumatic tap

Which of the following exhibits rapid motility in urine sediment

Trichomonas vaginalis

While performing the microscopic examination of the urine of a 20-year- old college student, the technician notes flagellar movement on several large cells with undulating membranes. She reports:

Trichomonas vaginalis

Which of the following cardiac markers is consistently increased in persons who exhibit unstable angina?

Troponin T

What is the recommended number of samples that should be tested to confirm a negative occult blood result

Two samples taken from different parts of three stools

Postural (orthostatic) proteinuria requires:

Two urine specimens

A 21-year-old female presents at the university clinic with complaints of increased thirst, urination, and hunger. She says that she eats constantly, yet she has lost about 10 pounds and her weight is down to 100 pounds. She is tired all the time, and her vision is a little blurry. She also reports that her urine smells "sort of fruity." Her blood pressure, temperature, and pulse all are normal at the time of examination. The patient's past clinical history reveals no previous disease conditions. Urinalysis and blood work are ordered. Her blood glucose level is 600 mg/dL, and the UA results are as follows:

Type 1 diabetes

The correct formula for calculating creatinine clearance is C (mL/min) =

U ×V/P 1.73m2/SA

The enzyme that conjugates bilirubin in the hepatocyte is:

UDP-glucuronyl transferase

Standard Precautions are a combination of major features of which two practice guidelines?

UP and BSI

The reciprocal plot pictured below indicates a decrease in Vmax and a decrease in Km. The solid line indicates a normal enzymatic reaction. What type of enzymatic inhibition is this

Uncompetitive

Substances that require elimination from the body by renal excretion include:

Urea, creatinine, and uric acid

A 36-year-old individual visits her physician with a complaint of nausea, loss of appetite, weakness, and an inability to concentrate. Laboratory results indicate increased serum urea and creatinine, increased potassium, reduced glomerular filtration rate (GFR), low blood pH, anemia (low red blood cell count), and hypocalcemia. What is the likely diagnosis?

Uremia

Why does persistently acidic urine lead to formation of uric acid renal stones?

Uric acid has a pKa of 5.57, which makes it insoluble in acidic solutions

Which of the following is exempt from decontamination by autoclaving or incineration before disposal?

Urine

Urine concentration measuring the number of solutes present can be assessed by:

Urine osmolality

Which factor contributes to urine color?

Urine pH

Water homeostasis is achieved by which of the following?

Urine volume relevant to hydration status and metabolic water production

"All of the following statements about urine specimen volume are true except:"

Urine volumes less than 3 mL must be rejected as QNS (quantity not sufficient).

Bilirubin is reduced to __________ in the small intestine.

Urobilinogen

All of the following can result in a turbid urine except:

Urochrome

________ are produced by the liver, are major carriers of endogenous triglycerides and can be responsible for the cloudiness of fasting hyperlipidemic plasma samples.

VLDL

Aspartate aminotranferase (AST) and alanine aminotransferase (ALT) are both elevated in which of the following diseases?

Viral hepatitis

The urine microscopic constituents that best differentiate between cystitis and pyelonephritis are

WBC casts

Principle solutes in the end-product of the urine filtrate include all of the following except:

Water

The following MUA results were obtained on a specimen from a sexually active 20-year-old college student who was admitted to the hospital with anxiety, abdominal pain, muscle pain, tingling, numbness, weakness, excessive sweating, and slight disorientation.

Watson-Schwartz test

Synovial fluid for crystal examination should be examined as a/an

Wet preparation

Which one of the following statements concerning lipase (LPS) is incorrect

When comparing LPS with amylase in diagnosis of acute pancreatitis, amylase is a more specific diagnostic finding

Why is bone disease a consequence of chronic kidney disease (CKD)?

When glomerular filtration declines, vitamin D activation decreases resulting in reduced calcium, which further leads to resorption of calcium from bone

An unpreserved specimen collected at 8 a.m. and remaining at room temperature until the afternoon shift arrives can be expected to have

"1 and 2 only Decreased glucose and ketones Increased bacteria and nitrite"

"If serum proteins were immersed in a buffer of pH 8.6, which protein, albumin (pI = 4.7) or gamma globulin (pI = 7.6) would migrate the farthest by electrophoresis, and toward which electrode?

"Albumin/anode

A urine that turns black after standing may contain

"All of the above Homogentisic acid, Melanin, Methemoglobin"

A creatinine clearance is done on a patient with the following results: serum creatinine, 0.6 mg/dL; urine creatinine, 102 mg/dL; urine volume, 1650 mL/24 hours; body surface area, 1.93 m2. Based on the information from calculations above, which of the following is the best interpretation of this patient's creatinine clearance results?

Above normal

Caffeine is used in bilirubin assays to

Accelerate indirect bilirubin reaction

The technician pulls a urine specimen out of the refrigerator and notes a pink precipitate. She knows that she has a urine with a(n):

Acidic pH and amorphous phosphates

Diseased renal tubules can put a patient at risk for which of the following?

Acidosis

Insulin may be described by all of the following EXCEPT:

Active in its C-peptide form

The countercurrent multiplier mechanism multiplies the osmotic gradient as a result of which of the following?

Active reabsorption of sodium and chloride and passive reabsorption of water

Large numbers of mitochondria and enzymatic activity in the proximal convoluted tubule and the loop of Henle are necessary for:

Active transport

A 10-year-old female presents to the ED with fever, malaise, nausea, oliguria, and edema of the eyes, knees, and ankles. Her blood pressure is mildly elevated. The creatinine clearance is decreased, with an elevated blood urea nitrogen to creatinine ratio. The clinical history reveals that the child had strep throat several weeks ago and that she appeared to have recovered without incident.

Acute glomerulonephritis

alpha-1 antitrypsin

Acute hepatic disease

Glomerular damage characterized by hematuria, proteinuria, oliguria, azotemia, edema, and hypertension is referred to as:

Acute nephritic syndrome

A 23-year-old female presents to the ED with sudden onset of flank and back pain, dysuria, and urgency. She is febrile and has chills, nausea, and headache. She reports no illnesses in the preceding weeks, and she currently is not taking any medica- tions. A urinalysis is ordered, and the results are as follows:

Acute pyelonephritis

In which liver disease is the DeRitis ratio (AST:ALT) usually less than or equal to 1?

Acute viral hepatitis

HBsAg PositiveAnti-HBc PositiveAnti-HBc (IgM) PositiveAnti-HBs Negative

Acutely infected

To achieve uniformity of technique by all those working in the urinalysis department, a laboratory requires all of the following except:

Adherence to procedure manual protocols

The reading of distilled water by the refractometer is 1.003. You should

Adjust the set screw

The UA of a 70-year-old male who takes an alkaline medication reveals 3+ protein on the reagent test strip and a pH of 9.0. The technician should:

Adjust the urine pH to 5.0 and retest using a reagent strip

Which of the following statements describes a nonkinetic enzyme assay?

"Initial absorbance is measured followed by a second reading after 5 minutes"

A nonspecific response to inflammation that includes elevation of certain plasma proteins and decrease of others due to cytokine production is referred to as a(n)

"acute phase reaction

4. A paraprotein is:

. a monoclonal immunoglobulin.

The percentage of cardiac output that flows through the kidneys per minute is:

0.3

Which of the following specific gravities would be most likely to correlate with a dark yellow urine

1.0

The normal reference range for urine specific gravity is:

1.002 to 1.035

A creatinine clearance is done on a patient with the following results: serum creatinine, 0.6 mg/dL; urine creatinine, 102 mg/dL; urine volume, 1650 mL/24 hours; body surface area, 1.93 m2. This patient's 24-hour creatinine excretion is

1.7 gm/day

In an individual with a normal hematocrit, whole-blood glucose values are approximately what percent different from plasma glucose values

10% lower

In most compound light microscopes, the ocular lens has a magnification of

10x

A CSF was hazy and the WBC was too high to perform undiluted. The technologist took 50 mL of sample and added 500 mL of saline. The cell count on the diluted sample was 200 WBC per mL. This should be multiplied by

11.0

Which urine specimen should be rejected?

12 mL specimen with a small amount of fecal matter

The normal glomerular filtration rate is

120 mL/min

Normal urine output averages how many milliliters per 24 hours?

1200.0

Reference Range for ALP

13-39

The normal value of CSF protein is

15-45 mg/dL

The normal renal threshold for glucose in the adult is approximately

160 mg/dL (8.8 mmol/L)

A serum specimen for glucose measurement was allowed to stand unseparated on the clot for 6 hours at room temperature before testing. If the original glucose concentration was 200 mg/dL, what approximate results would be expected after 3 hours?

160 mg/dl

A creatinine clearance is done on a patient with the following results: serum creatinine, 0.6 mg/dL; urine creatinine, 102 mg/dL; urine volume, 1650 mL/24 hours; body surface area, 1.93 m2. This patient's creatinine clearance, corrected for body surface area, is:

175 mL/min

A urine tested with Cliniteset exhibits a passthrough reaction and is diluted by adding 2 drops of urine to 10 drops water. This is dilution of

1:06

While running a second load of urine samples on the automated urinalysis instrument, a seasoned technician notes that all the patient samples are showing positive nitrates but no indication of bacteria upon microscopic examination. This is an example of which of the following?

2 and 3

A quantitative assessment of the urobilinogen level is performed on which type of urine specimen?

2-hour collection obtained between 2 and 4 PM

What is the globulin concentration of a specimen given the following information: total protein, 6.5 g/dL, and albumin, 3.8 g/dL?

2.7 g/dL

Which of the following results would be the most consistent with high risk for coronary heart disease?

20 mg/dL HDL cholesterol and 250 mg/dL total cholesterol

The LDL cholesterol for this patient would be ____________ mg/dL.

240.0

A 36-year-old female presents to the ED with shaking, chills, fever, and nausea, as well as abdominal, flank, and low and mid-back pain. A CAT scan and UA are ordered, along with other blood work. The CAT scan shows numerous kidney stones in the renal pelvis. The UA results reveal pathologic changes:

3 is correct

A patient's CK-MB is reported as 18 μg/L and the total CK is 560 IU/L. What is the CK relative index (CKI)?

3.2% (math)

The dimensions of a hemacytometer are

3x3x0.1 mm

Cholesterol crystals will most likely be observed in urine that contains

4+ protein

Rapid forward progression of sperm is rated as

4.0

What would the concentration of albumin be if the total protein value was 8.5 g/dL and the densitometer measured the albumin fraction as 55% of the total

4.68 g/dL

Centrifugation requirements for urine sediment preparation are:

450 g for 5 minutes

Many enzymes in the blood plasma show maximum activity in which of the following pH ranges?

7 to 8

Given the following information, calculate the sperm concentration: dilution, 1:20; sperm counted in five RBC squares on each side of the hemocytometer, 80 and 86; volume, 3 mL.

83 million per milliliter

Given the following information, calculate the creatinine clearance:24 hour urine volume 1100 ml; serum creatinine 1.5 mg/dL, urine creatinine 175 mg/dL

88.67 mL/min

What is the classic test to determine a diagnosis of hypoglycemia?

A 72-hour fast

State the classification of the below enzymes:A. Creatine kinaseB. Lactate dehydrogenaseC. Alkaline phosphatase

A TransferaseB OxidoreductaseC Hydrolase

Calibration of a refractometer requires the use of:

A and B

Neonatal screening tests are routinely performed for which condition to prevent mental retardation through dietary restrictions?

A and C

Which statement about enzymes is true?

A competitive inhibitor will alter the apparent Km of the reaction

Isosthenuria is a term used to describe

A fixed specific gravity of 1.010

What is the importance of the ionic strength of the buffer used in gel electrophoresis?

A heat-labile protein will become denatured if the ionic strength of the buffer is too high

You are examining a protein electrophoresis pattern and notice an unusual band at the starting point of the electropherogram that is not present in the normal control. If the patient were healthy, what is the most likely cause of the unusual banding pattern?

A plasma sample was probably applied instead of serum.

Match the enzyme with its correct substance required for enzyme reaction.A. Creatine kinaseB. Aspartate aminotransferase (AST)C .Amylase

A. PyridoxalphosphateB. MagnesiumC. Calcium

What statements concerning lipase (LPS) is incorrect?

A.When comparing LPS with amylase in diagnosis of acute pancreatitis, amylase is a more specific diagnostic finding.

Hypertonicity or hypotonicity of urine is controlled by:

ADH

Water reabsorption in the collecting tubules is controlled by:

ADH

A 65-year-old man has been diagnosed with prostate cancer and is being prepared for radiation therapy. His physician is concerned that the cancer may have metastasized to the patient's skeletal system. What laboratory analytes would give this information to the physician?

ALP

Which one of the following analytes is used as a marker for bone formation?

ALP

In viral hepatitis, the aminotransaminase enzyme that persists longer is ______, and the most liver specific enzyme is __________

ALT; ALT

Tumor markers that can be measured on body fluids include all except

ANA

Type 1 diabetes mellitus may be described by all of the following EXCEPT:

Adult-onset type

The reagent strip for protein can detect which protein(s)?

Albumin

Unconjugated bilirubin circulated in the bloodstream is bound to:

Albumin

Which one of the following statements concerning albumin is not correct

Albumin is involved in plasma reduction and oxidation reactions

"Which of the following enzymes is considered most tissue specific?"

Alcohol dehydrogenase (ADH)

A reddish brown urine specimen is received in the laboratory. The technician most likely will expect which reagent strip results?

Alkaline pH, positive blood

What is considered to be a nonspecific tumor-associated marker?

Alkaline phosphatase (ALP)

Heat inactivation and chemical inhibition are two methods used to measure which of the following analytes?

Alkaline phosphatase isoenzymes

Which of the following is the primary reagent used in the Jaffe reaction for creatinine?

Alkaline picric acid

What reagent is used in the bilirubin method to shift the color reaction to a wavelength with less interference

Alkaline tartrate

A routine urinalysis test can be used to diagnose what type of disease process?

All of the above

The ultimate goal of the kidneys is to:

All of the above

What proteins is the most abundant plasma protein in early embryonic life and is also measured to assess hepatocellular carcinoma in adults

Alpha-fetoprotein

Which one of the following proteins is the most abundant plasma protein in early embryonic life and is also measured to assess hepatocellular carcinoma in adults?

Alpha-fetoprotein

What stains are used to stain a serum protein electrophoresis gel

Amido Black and Coomassie Brilliant Blue

Fanconi's syndrome is a proximal tubular dysfunction that produces which of the following urine findings?

Aminoaciduria, proteinuria, phosphaturia, and glycosuria

The UA results for the patient in question 12 are positive for blood, but no RBCs are seen on the microscopic examination. The next step is perform a(n):

Ammonium sulfate precipitation test

You arrive in the UA department at 7 AM and find five routine urine samples in the refrigerator awaiting analysis. You pull the specimens out, check the dates and times, and let them warm to room temperature before analysis. You groan because you know that if a microscopic is required, the view may be obscured by:

Amorphous crystals

One extracellular enzyme whose serum activity is measured clinically is:

Amylase

Which of the following enzymes is activated by calcium ions

Amylase

The pathogenesis of which systemic disorder involves deposition of a pathologic proteinaceous substance between cells in numerous tissues and organs?

Amyloidosis

An 80-year-old woman is admitted to the hospital with disorientation, a temperature of 102° F, cough, congestion, and a preliminary diagnosis of pneumonia. She is catheterized, and a urine specimen is sent down for UA. The technician notes sheets of transitional epithelial cells in the urine sediment. The presence of these cells is most likely due to:

An iatrogenic process

The technician performs a microscopic examination on urine sediment from a 35-year-old female. The findings for this specimen are:

An insufficient amount of leukocyte esterase was produced.

The day shift technician pulled the UA reagent box out of the drawer and began performing QC using the following reagents: dipsticks, Clinitest, phenylalanine, Ictotest, and water in the dropper bottle. The technician is in violation of which quality assurance component?

Analytical

Secretion of renin and aldosterone is induced by low blood pressure and volume. What other hormone would be released in the event of low blood pressure and volume?

Antidiuretic hormone

The pathogenesis of glomerular damage marked by immune-mediated processes involves:

Antigen-antibody complexes

A semen specimen delivered to the laboratory in a condom has a normal sperm count and markedly decreased sperm motility. This is indicative of

Antispermicide in the condom

Which of the following apolipoproteins, when present in an increased concentration, would be associated with a decreased risk of coronary artery disease?

Apo AI

The glucose value of a NORMAL 2-hour-post-prandial serum specimen as compared to the reference range for a fasting serum glucose, should be:

Approximately the same as the normal fasting glucose level

In the countercurrent multiplier mechanism, the flow of the ultrafiltrate is in opposite directions between the:

Ascending and descending loops of Henle

The technician evaluates the following urinalysis results and notes a discrepancy between the reagent test and the microscopic examination for blood:

Ascorbic acid

Whenever the reagent pad shows a negative result for blood and the microscopic examination reveals RBCs, the technician should suspect the presence of:

Ascorbic acid

A 45-year-old man with polycystic disease is admitted to the hospital. The technician scans for casts and notes very large, colorless, hyaline-like casts that appear to have sharp, well-defined edges, some lateral fissures, and uneven ends. She checks the UA chemical analysis:

Broad, waxy casts

Which type of cast indicates significant urinary stasis and a poor prognosis?

Broad, waxy casts

The single most useful means of identifying a fluid as urine is to test:

Creatinine

Which clearance test results can be normalized using the patient's height, weight, and body surface area?

Creatinine clearance

In multiple myeloma, what protein is likely causing an elevated value in the urine protein confirmatory test?

Bence Jones protein

In the measurement of urea, urea is initially hydrolyzed by urease to form ammonium ions. This assay combined with the spectrophotometric measurement of the resulting ammonia is referred to as the:

Berthelot reaction

Amniotic fluid specimens are place in amber colored tubes prior to sending them to the laboratory to prevent the destruction of

Bilirubin

The normal composition of feces includes all of the following except

Blood

Failure to mix a specimen prior to inserting the reagent strip will primarily affect the

Blood reading

All of the following are improper uses of the reagent strip, which will produce erroneous results, except:

Blotting the edge of the reagent strip after dipping it in the urine

A physician orders several laboratory tests on a 55-year-old male patient who is complaining of generalized (all-over) pain, stiffness, fatigue, and headaches. Based on the following serum test results, what is the most likely diagnosis: ALP—significantly increased, GGT— normal?

Bone disease

An international unit (IU) of enzyme activity is the quantity of enzyme that:

Converts 1 μmol of substrate to product per minute

The exclusive site of plasma filtration is the:

Cortex

What transferase enzyme is involved in the conversion of the stored form of energy to usable energy (adenosine triphosphate [ATP]) within cardiac myocytes

Creatine kinase

An individual's laboratory results indicate increased serum creatinine and urea values, and his physician's diagnosis indicates a postrenal obstruction. What would the glomerular filtration rate in this individual likely be?

Decreased

A patient with elevated serum nitrogen compounds, markedly reduced GFR, increased serum sodium and potassium, and metabolic acidosis is diagnosed with acute kidney injury (AKI). Which one of the following is a likely cause?

Decreased cardiac output

Certain muscle relaxants are hydrolyzed by CHE during surgical procedures. If an individual with the atypical (Ea) gene variant is exposed to this relaxant, what would occur?

Decreased drug metabolism over time with resultant paralysis of respiratory muscles

In an individual with chronic kidney disease, what might be the predominant cause of the low hemoglobin value and anemia?

Decreased erythropoietin synthesis

What effect does epinephrine have on blood glucose value?

Decreases

What effect does ethanol have on blood glucose value?

Decreases

What effect does salicylate have on blood glucose value?

Decreases

What effect does septicemia have on blood glucose value

Decreases

Fanconi syndrome is a disease of

Defective proximal tubules

What is the presumed defect in most cases of familial type IIa hyperlipoproteinemia?

Defective receptors for LDL

Hyperchylomicronemia (type I) in childhood has been associated with which of the following?

Deficiencies of apolipoprotein CII and lipoprotein lipase

A type of brain damage that results from hyperbilirubinemia in an infant. Bilirubin is toxic to brain cells and easily crosses the blood-brain barrier to damage brain cells.

Kernicterus

It is important to keep the lids on urine specimens before testing so that volatilization does not occur for which urine constituent?

Ketones

The reference method for protein determination that converts nitrogen to ammonium ions by acid digestion is the

Kjeldahl method.

Regarding enzyme kinetics, the substrate concentration at which the reaction velocity is equal to 0.5 Vmax is referred to as:

Km

A severely hemolyzed serum specimen from an individual with hemolytic anemia is received in the laboratory with a request for the total CK and LD enzymes. How will the results of these analyses be falsely affected by the condition of the specimen?

LD increased, CK increased

A severely hemolyzed serum specimen from an individual with hemolytic anemia is received in the laboratory with a request for total CK and LD enzymes. How will the results of these analyses be falsely affected by the condition of the specimen

LD increased, CK increased

A notably hemolyzed serum specimen is received in the lab with a request for CK and LD enzymes and LD isoenzyme electrophoresis. How will the results of these analyses be falsely affected by the condition of the specimen?

LD increased; CK increased; Increased LD-1

Which of the following lipoproteins is the major carrier for cholesterol to peripheral tissues

LDL

Lactate dehydrogenase catalyzes the reaction of

Lactate --> pyruvate

UA results on a 28-year-old female are as follows: The technician notes the discrepancy between the chemical examination findings for LE and blood (i.e., negative) and the microscopic examination findings (i.e., presence of WBCs and RBCs). Which of the following may explain the discrepancy?

Specific gravity

Secretion of aldosterone stimulates the kidneys to retain:

Sodium and water

The preferred collection tube for preservation of glucose is:

Sodium fluoride

Measurement of which of the following is a means of monitoring the kidneys' ability to process and concentrate urine ultrafiltrate?

Sodium or chloride

The process of transferring the electrophoresed DNA fragments out of a gel and onto a special nylon paper is referred to as

Southern blotting

The most widely used method for bilirubin analysis uses two reagents, ____________ and ___________, in the diazo reaction.

Sulfanilic acid; sodium nitrite

Iatrogenic crystals include which of the following?

Sulfonamides

Collection of urine for anaerobic culture requires which of the following urine collection methods?

Suprapubic

HBsAg NegativeAnti-HBc NegativeAnti-HBs Negative

Susceptible

Which of the following is not considered potentially infectious and capable of disease transmission?

Sweat

The urine of a patient who had been on a medical fast for the past several weeks may smell:

Sweet and fruity

Secondary glomerular diseases include which types of disorders?

Systemic

Cost-effective practices in the laboratory include which of the following?

Test utilization

All casts typically contain

Tamm-Horsfall glycoprotein

An individual visits a physician, who notes that the patient has splenomegaly, enlargedand orange-colored tonsils, and tingling feelings in the extremities, indicating peripheral neuropathy. Upon examination of the blood smear for a white blood cell differential, many foamy-appearing cells were noted. Normal triglyceride and total cholesterol values were noted; however, a marked decrease of HDL was also noted. What is the name of this disorder?

Tangier disease

The ability to resolve discrepant test results in the laboratory is an example of:

Technical competence

A client comes to the outpatient department for a urine drug screen. You follow the protocol for collection but question whether the specimen is truly urine. You then follow the procedure for urine verification and obtain the following results. Which result causes you to reject the specimen?

Temperature of 30° C

Which factor is a consideration in the collection of urine for creatinine clearance measurement?

Temperature sensitivity

A 23-year-old female patient comes in for a urinalysis on a midstream clean catch specimen. The results are as follows:

The results are inaccurate, because a discrepancy exists between the chemical and microscopic results.

Pleural fluid is collected by

Thoracentesis

The means of preservation used to preserve urine sediment elements is:

Thymol

The patient comes in with an order for a creatinine clearance test. The technician will instruct the patient to collect which specimen type?

Timed

Quantitative urine tests are performed on

Timed specimens


Kaugnay na mga set ng pag-aaral

NHW Elementary, Third edition, SB, Unit 13, p. 100 - adjectives and adverbs

View Set

Computer Applications - Ch. 14 Excel

View Set

Chapter 27 Development and Inheritance

View Set

(Ch. 3) Texas Principles of Real Estate Part 2

View Set

Angles of Elevation and Depression

View Set

Forensic Psychology Week 5 - Psychology of Violence and Intimidation

View Set